Orthogonal matrices form a group

Click For Summary
SUMMARY

The set of all n x n orthogonal matrices forms a group under matrix multiplication. This is established through the verification of group properties: closure, associativity, the existence of an identity element (the identity matrix), and the existence of inverses (the inverse of any orthogonal matrix is also orthogonal). The proof utilizes the properties of orthogonal matrices, specifically that the transpose of an orthogonal matrix is its inverse, ensuring that the product of two orthogonal matrices remains orthogonal.

PREREQUISITES
  • Understanding of orthogonal matrices and their properties
  • Knowledge of matrix multiplication and its associative property
  • Familiarity with the concept of identity matrices
  • Basic understanding of matrix inverses
NEXT STEPS
  • Study the properties of orthogonal matrices in linear algebra
  • Learn about group theory and its applications in mathematics
  • Explore the significance of matrix inverses in transformations
  • Investigate the role of orthogonal matrices in computer graphics and data science
USEFUL FOR

Students of linear algebra, mathematicians studying group theory, and professionals in fields such as computer graphics or data science who require a solid understanding of matrix operations and transformations.

spaghetti3451
Messages
1,311
Reaction score
31

Homework Statement



Show that the set of all ##n \times n## orthogonal matrices forms a group.

Homework Equations



The Attempt at a Solution



For two orthogonal matrices ##O_{1}## and ##O_{2}##, ##x'^{2} = x'^{T}x' = (O_{1}O_{2}x)^{T}(O_{1}O_{2}x) = x^{T}O_{2}^{T}O_{1}^{T}O_{1}O_{2}x = x^{T}O_{2}^{T}O_{2}x = x^{T}x = x^{2}.##

So, closure is obeyed.

Matrix multiplication is associative.

The identity element is the identity matrix.

##x'^{2} = (O^{-1}x)^{T}(O^{-1}x) = x^{T}(O^{-1})^{T}O^{-1}x = x^{T}(O^{T})^{-1}O^{-1}x = x^{T}(OO^{T})^{-1}x = x^{T}x = x^{2}##.

So, the inverse of any orthogonal matrix is an orthogonal matrix.

Is my answer correct?
 
Physics news on Phys.org
Looks good to me
 
Question: A clock's minute hand has length 4 and its hour hand has length 3. What is the distance between the tips at the moment when it is increasing most rapidly?(Putnam Exam Question) Answer: Making assumption that both the hands moves at constant angular velocities, the answer is ## \sqrt{7} .## But don't you think this assumption is somewhat doubtful and wrong?

Similar threads

  • · Replies 7 ·
Replies
7
Views
8K
  • · Replies 3 ·
Replies
3
Views
2K
Replies
6
Views
2K
  • · Replies 3 ·
Replies
3
Views
2K
  • · Replies 4 ·
Replies
4
Views
2K
  • · Replies 2 ·
Replies
2
Views
1K
  • · Replies 2 ·
Replies
2
Views
2K
  • · Replies 3 ·
Replies
3
Views
2K
  • · Replies 8 ·
Replies
8
Views
3K
Replies
11
Views
2K